I don't totally trust to some argument in theories about The Earth Magnetism. That's why I keep researching.
I find interested with this forum, that's why I join. Although I join to offer nothing at the moment, but hope to offer something in future. Currently I enjoy with reading your...
I am told that in ferromagnets, time reversal symmetry is broken. However, I don't know any hamiltonian terms in solid that can break time reversal symmetry. So is there a hamiltonian term I don't know or is there any subtlety in ferromagnets?
Guys, I'm having trouble with some concepts on the Biot-Savart Law. Lots of texts compare this law with Coulomb Law, but doesn't affirm that B-S is only applicable on a static case (constant current). What are the real physical conditions in which Biot-Savart Law can be directly applied? Does it...
Homework Statement
Hello everyone, I am new here and have a question regarding method of images in my electricity and magnetism class. I need help to even get the ball rolling. The question is as follows:
a) What is the image of a dipole, oriented toward the center of the conducting sphere, if...
Homework Statement
I'm working on a problem that says that
>If $\varepsilon_{\pm}=\mp (\mu \mu_0 H + k \theta \frac{M}{\mu N} )$ is the energy of the atom of a specimen that can orient itself either parallel or antiparallel in a magnetic field, show that $ \frac{M}{\mu N}= tanh(\frac{1}{kT}(...
Hi, I'm working with a magnetic levitron project and I've borrowed a professional levitron to see how it works. When I remove the back cap, unexpectedly I saw a square shaped magnet instead of ring magnet. But it acts like almost a ring magnet. I'll explain the difference with images below.
At...
Homework Statement
A square with each side of 5 cm in length.Now if 4 parallel wires in each 4A current is flowing were placed on the vertex of the square.How can I find the center of the square of the magnetic strength?
Homework Equations
I am not sure what equation should be used.If I knew I...
Homework Statement
For a Stern-Gerlach experiment, there is a apparatus designed to create a magnetic gradient. There is a dipole magnet. The radius of the convex pole is 5 cm, the radius of the concave pole is 10 cm. The convex pole as a 2 T magnetic field along its surface. (The apparatus is...
Homework Statement
In a uniform magnetic field with induction of 0.1 Teslas - a coil is located perpendicular to the lines of induction (I suppose it's something like a ring of wire, meaning N=1). Resistance = 2 Ohms. What is the area of the "ring" if when the field is switched on - it will...
Curie temperature for magnetite is 858 C, for iron 1043 C. Duralumin melting point is about 630 C. Does this mean that if I take a small silicone mold, place a magnet underneath it, add ferromagnetic powder to the mold and then pour molten aluminum in, I will get the aluminum detail with...
Hello,
i've recently found the post
"How to calculate the force acting on a magnet placed in a magnetic field?"
at
http://physics.stackexchange.com/questions/20258/how-to-calculate-the-force-acting-on-a-magnet-placed-in-a-magnetic-field
in another Forum
and the answer said smth about an...
Hi, I'm an hobbyist
right now I'm into electric engines, simple engines for low voltage and low current applications; I have noticed that there are many variations about how and what an electric engine can do even considering only DC motors, but basically all the engines that I have examined up...
In chapter 5 of Purcell's Electricity and Magnetism, he shows using principles of special relatives that the magnetic force of a current carrying wire on a moving charge is equivalent to the charge experiencing an electric force in its own frame of reference.
His argument is that when the...
I'm reading how a charged particle moving through a magnetic field experiences a force (called magnetic force).
I understand that it moves the particle in a certain direction and because the greatest force is experienced when the magnetic field is perpendicular to the particle's velocity vector...
The 4 questions have been uploaded as 2 jpeg files. I need step by step proper solutions as a pdf or word file if possible. Please help.
Send the solutions to my email id : pyplx7x@gmail.com
Homework Statement
A flexible wire loop has a radius of r = 0.178 m and it is inside a uniform magnetic field of B = 0.332 T. The loop is grasped at points P and Q and stretched until its area is zero. It takes 0.146 s to close the loop.
Homework Equations
EMF = ∆Phi/∆t
Phi = magnetic flux...
In reading my physics book, there was mention of different types of poles. I was hoping someone could explain the differences and possibly supply a visual representation of the poles with respect to each other as to where they lie on Earth. The different poles were the magnetic N/S, geographic...
1. The magnetic field at point P due to a magnetic source S1 is represented by ==>. Can a bar magnet S2 be brought close to P so that the total magnetic field at P due to S1 and S2 is zero? Explain your answer.2. None3. Yes, this is possible if both fields at point P have the same magnitude but...
Homework Statement
An electric current is uniformly distributed throughout a long, straight wire that has a diameter of 0.05m. If the current through the wire is 6.0A, calculate the magnitude of the magnetic field:
a). 0.02m radially away from the wire center
b). 0.05m radially away from the...
I want to know that are there any materials or methods by which their relative magnetic permeability (μr) be instantaneously,continuously & periodically varied. Kindly provide the reference document/video for the description if possible.
Homework Statement
The problem is in the thumbnail
Homework Equations
None
The Attempt at a Solution
I just wanted to double check my answer on this:
Left side of the coil is North
Right side of the coil is South
The arrows that indicate the direction of the current should be facing up in...
hi, i know this question is a bit unnecessary but i just want to know if i got my concept right
so, here's the question
if you have a compass pointing northwards placed between two parallel wires carrying opposite current, does the needle deflect?
my answer is the needle won't deflect since the...
Homework Statement
This is for a practice question on an exam:
I am able to finish the problem, if I could figure out how to find the radius of this arc the proton makes.
Homework Equations
I have nothing.
The Attempt at a Solution
I have tried arc length equations and just integrating the...
Homework Statement
Homework Equations
Biot-Savart Law: ## dB = μIdl/4πr^2
F = ILxB
The Attempt at a Solution
I have not tried to solve it. BUT, please check my approach. I want to make sure my method is correct and whether there is a flaw or no.
I am trying to set up an integral here. I am...
Im trying to implement the implicit Euler method in high-performance software for micromagnetic simulations, where I'm restricted to using the driving function of the ODE (Landau-Lifshitz equation) and the previous solution points. This obviously not a problem for an explicit method, since we...
Okay guys I have an hour and thirty minutes before my exam; I know how to do the top problem; just use the formula B = mew x current/(2 x pi x r) then add the magnetic fields and use the right-hand rule to get direction, but I'm stuck on the bottom one.
Homework Statement
Suppose a subatomic particle of mass m kg has kinetic energy K.E. a nJ and is moving southward toward a vertical wall. When the particle is distance d cm from the wall a magnetic field is turned on: it has magnitude b T and points upward. The particle turns westward so it...
So, I was watching PBS's Nazi Mega Weapons, and they were discussing the V-1 terror drone when they said something that made me scratch my head. They said that the Nazis initially had problems with the compass that facilitated navigation because of magnetic interference from the metal in the V-1...
Hey guys,
I have a question. I am doing some homework for university and got super confused (probably just messing up things). So in school you learn that magnetic monopols do not exist. But my question is now: On the sun there are coronal holes, region where the magnetic field is not able to...
Imagine an infinitely long wire carrying current i1 in west to east direction. At a small distance 'd' above the wire there is another small current carrying wire of length 'l' carrying current i2 from east to west direction (opposite to the direction of current in the below placed wire)...
Hello everyone.
I am glad I came across this science oriented Forum site.
I actually get there from Google and what I am deeply curious to know about is the amount of Maximum Voltage my 3.5kg Air Core Electromagnet can bear.
The Core of the Solenoid is 120mm in height and 13inches in...
Homework Statement
This is a question me and my friend were wondering about. How can one calculate the magnetic field due to a current carrying loop at a point in the area enclosed by the loop.
For example, at point P as shown in the attached figure.
Homework Equations
$$B =...
Hi, I'm a retired (since 2011) Physics prof from the University of British Columbia. I originally set out to get a PhD in Physics to increase my credibility as a science fiction writer, but I discovered a field* that was so cool it was like being a character in my own SF novel. In short, I got...
Homework Statement
I'm looking at an E&M textbook - "Time-Harmonic Electromagnetic Fields". They state:
"A more general ##x ## polarized field is one consisting of waves traveling in opposite directions with unequal amplitudes - i.e :
(1) $$ E_{x} = Ae^{-jkz} +Ce^{jkz}$$
Let ## A ## and...
Not a homework question! I am doing exercises for upcoming final exam.
So, I get stuck at question 5.27 (Griffith 4th edition textbook).
Question:
Find the vector potential above and below an infinite uniform surface current with constant current sheet, K flowing at positive x direction.
I...
Edit: sorry i realized i posted this in the wrong section. Can some move it to hw? Thank you.
Hey i was studying intro E&M recently and i started learning about RC circuits. Up until this point everything, concept wise, has made sense for me. Then I learned how RC circuits work, and the math...
CASE I - Do magnetic field is felt by observer which is moving and charge is stationary?
CASE II- Do magnetic field is felt by observer which is moving with same velocity as that of charge?
Does it has some experimental proofs and explanations...
I took a college physics course (Mechanics) at a CC, and my physics prof was notoriously known to be one of the best... many of the students transferred to Berkeley and say that he is one of the best they have taken. He said that his two favorite book are: Halliday Resnik: Fundamentals of...
The force on a particle moving through a magnetic field is given by:
$$\mathord{\buildrel{\lower3pt\hbox{$\scriptscriptstyle\rightharpoonup$}}\over
F} = q\,\mathord{\buildrel{\lower3pt\hbox{$\scriptscriptstyle\rightharpoonup$}}\over
v} \times...
Hi, I'm looking for some great books clearly explaining how electricity, magnetism and electromagnetism Works.
I'm also looking for some books describing how it was discovered and what experiments have been performed in discovering how it Works,.
So Far I've looked at
Electromagnetism...
Hi, I was wondering whether it was posible with a formula to measure the distance to which two equal magnets start attracting to each other. I was looking at this web page https://www.kjmagnetics.com/blog.asp?p=surface-fields-101 and I am not sure if this is the right formula.
I have two...
I know this question has been beaten to death, but I haven't seen a response that clearly (to me) answers the following:
1. Magnetic fields *can do work* on intrinsic dipoles, right? (e.g. two electrons can do work on one another via their intrinsic spin).
2. Magnetic materials can do work on...
Homework Statement
I came across a recent problem that asked me to calculate the flux density at the end of a solenoid. I was given the current 3 A , the number of turns per unit length , 12 cm-1 and the using the permeability of free space as 4π × 10^-7
Homework Equations
The equation I used...
Q1.In school, we were learning about the magnetic properties of solids, and my teacher mentioned this, a spinning electrons create magnetic field. I cannot understand how. Like earlier we were told if a charge is in motion, it creates magnetic field, but in spinning, it does not have...
A Science Daily report discusses observations of cosmic gamma rays that suggest that magnetic fields in the universe disproportionately have left handed rather than right handed helicity. It goes on to suggest that under a 2001 theory of another physicist that this could explain...
Hello, I've been having some trouble with a paramagnetism problem from my Statistical Mechanics class textbook (F. Mandl, Statistical Physics, 2nd edition, p. 25). The problem is as follows
1. Homework Statement
2. Homework Equations
1. The temperature parameter
\displaystyle{ \beta =...